- PowerScore Staff
- Posts: 5972
- Joined: Mar 25, 2011
- Wed Apr 22, 2020 9:55 am
#88169
Complete Question Explanation
(The complete setup for this game can be found here: lsat/viewtopic.php?f=170&p=88166#p88166)
The correct answer choice is (D).
A brief glance at the answer choices reveals that the form of the answers is somewhat unique: each answer specifies that the CD in question is the “only” CD to receive the specified rating. Although we will examine the distribution in a moment, the best starting point in a question of this type is to refer to the work done in previous questions:
Answer choice (A) can be disproved by the work done in question #9, which revealed that Q could be the only CD to receive three stars.
Answer choice (B) can be disproved by the work done in questions #8 and #9, which revealed that Q could be the only CD to receive four stars.
Answer choice (C) can be disproved by the question stem in question #11, which shows the value of waiting until the end of the game to attack Global questions.
Thus, by referring to questions #8 and #9, we can eliminate answer choices (A) and (B). If you did not wait until the end of the game to come back to this question, and answer choice (C) is still in contention for you, do not despair. Answer choices (C), (D), and (E) each involve R being the only CD to receive a particular rating. Thus, let’s examine what occurs when R is the only CD to receive a certain rating:
(The complete setup for this game can be found here: lsat/viewtopic.php?f=170&p=88166#p88166)
The correct answer choice is (D).
A brief glance at the answer choices reveals that the form of the answers is somewhat unique: each answer specifies that the CD in question is the “only” CD to receive the specified rating. Although we will examine the distribution in a moment, the best starting point in a question of this type is to refer to the work done in previous questions:
Answer choice (A) can be disproved by the work done in question #9, which revealed that Q could be the only CD to receive three stars.
Answer choice (B) can be disproved by the work done in questions #8 and #9, which revealed that Q could be the only CD to receive four stars.
Answer choice (C) can be disproved by the question stem in question #11, which shows the value of waiting until the end of the game to attack Global questions.
Thus, by referring to questions #8 and #9, we can eliminate answer choices (A) and (B). If you did not wait until the end of the game to come back to this question, and answer choice (C) is still in contention for you, do not despair. Answer choices (C), (D), and (E) each involve R being the only CD to receive a particular rating. Thus, let’s examine what occurs when R is the only CD to receive a certain rating:
When R is the only CD to receive a particular rating, then from the third rule we can infer that H and I must receive the same rating, with H receiving one more star than N:
Although Q and S have yet to be considered, the information above is actually sufficient to solve this question. Because I can never receive four stars (as discussed in the setup), this block has only two placement options: 1-2 and 2-3. Accordingly, either N or the HI block always receive two stars. As R must be the only CD to receive its rating, R can therefore never receive two stars, and answer choice (D) is correct.
You do not have the required permissions to view the files attached to this post.
Dave Killoran
PowerScore Test Preparation
Follow me on X/Twitter at http://twitter.com/DaveKilloran
My LSAT Articles: http://blog.powerscore.com/lsat/author/dave-killoran
PowerScore Podcast: http://www.powerscore.com/lsat/podcast/
PowerScore Test Preparation
Follow me on X/Twitter at http://twitter.com/DaveKilloran
My LSAT Articles: http://blog.powerscore.com/lsat/author/dave-killoran
PowerScore Podcast: http://www.powerscore.com/lsat/podcast/